LSAT Logic Games : LSAT Logic Games

Study concepts, example questions & explanations for LSAT Logic Games

varsity tutors app store varsity tutors android store

Example Questions

1 2 70 71 72 73 74 75 76 78 Next →

Example Question #261 : Solving Grouping Games

Alpha Corporation's Board of Directors (the "Board") consists of exactly five Directors, one of whom is designated as the Chairperson. The Directors are chosen from among a group of five executives of Alpha Corporation - A, B, C, D, and E - and a group of four people not otherwise associated with the company - W, X, Y, and Z. All choices must be made in accordance with the following conditions:

The Board may not include fewer than two Directors from each group.

The Chairperson must be a Director chosen from a group from which exactly one other Director is chosen.

If X is not chosen, A may not be chosen.

If D is not chosen, B may not be chosen.

If either C or E is chosen, the other must also be chosen.

E and W cannot both be chosen.

If A is designated as the Chairperson, which one of the following must also be chosen to be a Director?

Possible Answers:

Y

W

D

B

Z

Correct answer:

D

Explanation:

If A is Chairperson, only one other Director can be chosen from {B, C, D, E} (Condition 2). However, neither C nor E can be that Director because choosing one requires choosing the other (Condition 5). Similarly, B cannot be chosen because then D would also have to be chosen (Condition 4). So, D must be the second company executive chosen to be a Director.

Further, since A is chosen, X must also be chosen (Condition 3), leaving two open spots for W, Y, and Z. However none of W, Y, or Z must be chosen (any 2 will do). So, D is the only listed person that must be chosen.

Example Question #771 : Lsat Logic Games

Alpha Corporation's Board of Directors (the "Board") consists of exactly five Directors, one of whom is designated as the Chairperson. The Directors are chosen from among a group of five executives of Alpha Corporation - A, B, C, D, and E - and a group of four people not otherwise associated with the company - W, X, Y, and Z. All choices must be made in accordance with the following conditions:

The Board may not include fewer than two Directors from each group.

The Chairperson must be a Director chosen from a group from which exactly one other Director is chosen.

If X is not chosen, A may not be chosen.

If D is not chosen, B may not be chosen.

If either C or E is chosen, the other must also be chosen.

E and W cannot both be chosen.

If A is chosen to be a Director, any of the following could be the Chairperson EXCEPT:

Possible Answers:

Z

W

X

D

C

Correct answer:

C

Explanation:

If C is chosen, E must also be chosen (Condition 5). However, that would put 3 company executives on the Board (A, C, and E), which means designating C as the Chairperson would violate Condition 2.

Example Question #261 : Solving Grouping Games

Alpha Corporation's Board of Directors (the "Board") consists of exactly five Directors, one of whom is designated as the Chairperson. The Directors are chosen from among a group of five executives of Alpha Corporation - A, B, C, D, and E - and a group of four people not otherwise associated with the company - W, X, Y, and Z. All choices must be made in accordance with the following conditions:

The Board may not include fewer than two Directors from each group.

The Chairperson must be a Director chosen from a group from which exactly one other Director is chosen.

If X is not chosen, A may not be chosen.

If D is not chosen, B may not be chosen.

If either C or E is chosen, the other must also be chosen.

E and W cannot both be chosen.

If neither A nor D is chosen to be a Director, which one of the following is possible?

Possible Answers:

W is a Director.

X is the Chairperson of the Board.

C is the Chairperson of the Board.

B is a Director.

Z is the Chairperson of the Board.

Correct answer:

C is the Chairperson of the Board.

Explanation:

If neither A nor D is a Director, B cannot be a Director (Condition 4), so C and E must both be Directors (Condition 1). Further, since C and E will be the only company executives chosen to be Directors, one must be designated as the Chairperson (Condition 2). So, C can be the Chairperson.

Example Question #265 : Solving Grouping Games

A new grocery store is deciding which brands of candy bars to sell in its opening stock. The store will select exactly five of the following eight brands to stock: Brighter, Charm, Delico, Flare, Handy, Joyful, Little, and Prospine. The selection of brands must follow these conditions:

If both Charm and Flare are selected, Delico is also selected.

If Handy is selected, neither Brighter nor Flare is selected.

If Little is selected, Joyful is not selected.

Of the brands Brighter, Joyful, and Prospine, exactly two are selected.

Which of the following could be a complete and accurate list of the candy bar brands selected for the store?

Possible Answers:

Brighter, Charm, Delico, Handy, Prospine

Brighter, Charm, Flare, Little, Prospine

Charm, Delico, Handy, Joyful, Prospine

Brighter, Delico, Flare, Joyful, Prospine

Charm, Delico, Joyful, Little, Prospine

Correct answer:

Charm, Delico, Handy, Joyful, Prospine

Explanation:

The correct answer is achieved by process of elimination. One answer is incorrect because it breaks the global rule that Little cannot be selected with Joyful. One answer is incorrect because it breaks the global rule that if Handy is selected, Brighter is not selected. One answer is incorrect because it breaks the global rule that if Charm and Flare are selected, Delico must also be selected. One answer is incorrect because it breaks the global rule that only and exactly two of Brighter, Joyful, and Prospine may be selected.

Example Question #772 : Lsat Logic Games

A new grocery store is deciding which brands of candy bars to sell in its opening stock. The store will select exactly five of the following eight brands to stock: Brighter, Charm, Delico, Flare, Handy, Joyful, Little, and Prospine. The selection of brands must follow these conditions:

If both Charm and Flare are selected, Delico is also selected.

If Handy is selected, neither Brighter nor Flare is selected.

If Little is selected, Joyful is not selected.

Of the brands Brighter, Joyful, and Prospine, exactly two are selected.

If Little is selected, which one of the following is a pair of brands both of which must also be selected?

Possible Answers:

Flare and Delico

Charm and Prospine

Handy and Brighter

Brighter and Flare

Prospine and Brighter

Correct answer:

Prospine and Brighter

Explanation:

Since you know Little is selected, you know that Joyful is not selected, based on the third global rule. This then allows you to conclude that Prospine and Brighter must be selected, based on the fourth global rule. Since Joyful cannot be selected, Prospine and Brighter must be selected to fulfill that rule.

Example Question #264 : Solving Grouping Games

A new grocery store is deciding which brands of candy bars to sell in its opening stock. The store will select exactly five of the following eight brands to stock: Brighter, Charm, Delico, Flare, Handy, Joyful, Little, and Prospine. The selection of brands must follow these conditions:

If both Charm and Flare are selected, Delico is also selected.

If Handy is selected, neither Brighter nor Flare is selected.

If Little is selected, Joyful is not selected.

Of the brands Brighter, Joyful, and Prospine, exactly two are selected.

If the store does not select Brighter, which one of the following must be true?

Possible Answers:

Charm is selected.

Delico is not selected.

Handy is not selected.

Little is selected.

Flare is selected.

Correct answer:

Charm is selected.

Explanation:

Since Brighter is not selected, based off the fourth global rule, Joyful and Prospine must be selected. Since Joyful must be selected, Little cannot be selected. So far you are able to eliminate "Little is selected."

Two spots are now filled (Joyful, Prospine) and two are eliminated (Brighter, Little). That means you have four possible options for the three remaining spots (Charm, Delico, Flare, and Handy).

Based off the second global rule, Handy and Flare cannot be selected together, which tells you exactly one of them is selected and the other is not selected. No matter which is selected, you are left with two open spots and only two options: Charm and Delico. So, Charm and Delico must be selected. You have an answer choice that reflects this: "Charm is selected."

Example Question #265 : Solving Grouping Games

A new grocery store is deciding which brands of candy bars to sell in its opening stock. The store will select exactly five of the following eight brands to stock: Brighter, Charm, Delico, Flare, Handy, Joyful, Little, and Prospine. The selection of brands must follow these conditions:

If both Charm and Flare are selected, Delico is also selected.

If Handy is selected, neither Brighter nor Flare is selected.

If Little is selected, Joyful is not selected.

Of the brands Brighter, Joyful, and Prospine, exactly two are selected.

If both Flare and Joyful are selected, which of the following is a pair of brands both of which could also be selected?

Possible Answers:

Flare and Little

Charm and Prospine

Delico and Handy

Brighter and Handy

Charm and Little

Correct answer:

Charm and Prospine

Explanation:

Since you know Flare is selected, based off the second global rule, you know that Handy is not selected (by the contrapositive, if Flare is selected, Handy is not selected). This allows you to eliminate the two answer choices that include Handy.

Also, since you know Joyful is selected, based off the third global rule, you know that Little is not selected. This allows you to eliminate the two answer choices that include Little.

You are left with the correct answer: Charm and Prospine. You can verify this by filling the two spots with Charm and Prospine, which leads to the final spot being filled by Delico (via the first global rule).

Example Question #269 : Solving Grouping Games

A new grocery store is deciding which brands of candy bars to sell in its opening stock. The store will select exactly five of the following eight brands to stock: Brighter, Charm, Delico, Flare, Handy, Joyful, Little, and Prospine. The selection of brands must follow these conditions:

If both Charm and Flare are selected, Delico is also selected.

If Handy is selected, neither Brighter nor Flare is selected.

If Little is selected, Joyful is not selected.

Of the brands Brighter, Joyful, and Prospine, exactly two are selected.

Which one of the following brands must be selected?

Possible Answers:

Delico

Handy

Joyful

Little

Charm

Correct answer:

Delico

Explanation:

You could go through the possible answer choices, one by one, and check to see which brand can never be left out of a valid selection group. This, however, can be very time consuming.

One strategy is to use the work you have already done on previous questions. From Question 1, you have already verified a valid group is: Charm, Delico, Handy, Joyful, and Prospine. This allows you to eliminate "Little," as it is not always needed.

From Question 3, you discovered a possible outcome in which you must choose Flare or Handy, meaning it is possible to have an outcome that leaves out either one. This allows you to eliminate Handy for this question. You are now left with Charm, Delico, or Joyful.

The next step is to create a group that leaves out at least one of these, to prove that it is not needed always. If you start by leaving out Charm, you can create a hypothetical group that includes Flare, Delico, Brighter, Prospine, and Little. This is a valid group, allowing you to eliminate Charm. This group also happens to leave out Joyful, allowing you to eliminate it as a possible answer.

You are then only left with Delico as the correct answer.

Example Question #773 : Lsat Logic Games

A new grocery store is deciding which brands of candy bars to sell in its opening stock. The store will select exactly five of the following eight brands to stock: Brighter, Charm, Delico, Flare, Handy, Joyful, Little, and Prospine. The selection of brands must follow these conditions:

If both Charm and Flare are selected, Delico is also selected.

If Handy is selected, neither Brighter nor Flare is selected.

If Little is selected, Joyful is not selected.

Of the brands Brighter, Joyful, and Prospine, exactly two are selected.

If both Brighter and Prospine are selected, which one of the following is a pair of brands neither of which could be selected?

Possible Answers:

Handy and Joyful

Charm and Joyful

Flare and Little

Joyful and Little

Charm and Handy

Correct answer:

Handy and Joyful

Explanation:

Since Brighter is selected, based on the contrapositive of the second global rule, Handy cannot be selected (if Flare is selected, Handy is not selected). Since Brighter and Prospine are both selected, based on the fourth global rule, Joyful cannot be selected.

There is an answer choice that reflects these two that cannot be selected: Handy and Joyful.

Example Question #271 : Solving Grouping Games

The chef at a La Maison, a popular French restaurant, is famous for making nine different dishes: three beef dishes - A, B and C; three poultry dishes - L, M and N; and three fish dishes - X, Y and Z. La Maison's special tasting menu always consists of exactly five of its chef's famous dishes. Each month, the chef changes the dishes on the tasting menu, choosing from among his famous dishes in accordance with the following conditions:

There must be exactly one fish dish on the menu.

If Z is not on the menu, A cannot be on the menu.

No more than two beef dishes can be on the menu.

If both L and Y are on the menu, B cannot be on the menu.

If A is on the menu, N cannot be on the menu.

Unless both C and X are on the menu, M cannot be on the menu.

Which of the following dishes can never be chosen for any month's tasting menu?

Possible Answers:

B

A

M

X

L

Correct answer:

A

Explanation:

If A is chosen, N CANNOT be chosen (fifth condition) and neither can both of B and C (third condition).

Further, if A is chosen, then Z must also be chosen (second condition) and, thus, neither X nor Y can be chosen (first condition), which means M cannot be chosen (last condition).

So, we have deduced that if A is chosen, there are five dishes that cannot also be chosen (M, N, X, Y and one of B or C), leaving only three dishes that could possibly be chosen with A (for a total of four dishes on the menu, including A). However, each menu must have exactly five dishes, so A can never be chosen.

1 2 70 71 72 73 74 75 76 78 Next →

Tired of practice problems?

Try live online LSAT prep today.

1-on-1 Tutoring
Live Online Class
1-on-1 + Class
Learning Tools by Varsity Tutors